LSAT and Law School Admissions Forum

Get expert LSAT preparation and law school admissions advice from PowerScore Test Preparation.

User avatar
 Dave Killoran
PowerScore Staff
  • PowerScore Staff
  • Posts: 5848
  • Joined: Mar 25, 2011
|
#84838
Complete Question Explanation
(The complete setup for this game can be found here: lsat/viewtopic.php?t=5869)

The correct answer choice is (A).

The question stem adds the condition that all stations must be connected by two or more lines. Thus, examine the map for the stations currently connected by only one line:

F: connected currently only by line 1

P: connected currently only by line 1

G: connected currently only by line 4

Thus, the correct answer must include all three of the stations above, and consequently answer choice (A) is correct.

Get the most out of your LSAT Prep Plus subscription.

Analyze and track your performance with our Testing and Analytics Package.